Guess what I learned today
by v_Enhance, Jan 2, 2012, 6:59 AM
The following took 40 minutes.
(USAMO 2001.2) Let
be a triangle and let
be its incircle. Denote by
and
the points where
is tangent to sides
and
, respectively. Denote by
and
the points on sides
and
, respectively, such that
and
, and denote by
the point of intersection of segments
and
. Circle
intersects segment
at two points, the closer of which to the vertex
is denoted by
. Prove that
.
Click to reveal hidden text
(USAMO 2001.2) Let





















Click to reveal hidden text
So we use barycentric coordinates.
It's obvious that un-normalized,
, so we get a normaized
. Similarly,
.
Okay, so then we get the point
by intersecting the lines
and
.
Let
be such that
. It's obvious that
, so we find that $Q'_y = \frac{s-b}{a} - \frac{s-b}s} = \frac{(s-a)(s-b)}{sa}$. Also, since it lies on the line
, we get that
. Hence, ![\[ Q'_x = 1 - \frac{((s-b)+(s-c))(s-a)}{sa} = \frac{sa - a(s-a)}{sa} = \frac{a}{s} \]](//latex.artofproblemsolving.com/d/f/5/df5679b94f5164297ee9e522f771170be250e4a2.png)
Let
. Then the displacement vector of
is
which we want to show has length
. Let
and
. Using the inversion distance formula, as well as
, this becomes equivalent to:
But we also know that
by Heron, and ![\[ k=\frac{1}{2} (b^2-a^2-c^2). \]](//latex.artofproblemsolving.com/5/7/3/573e96c106d6c2883ed7ecc1c3a5511a7e740e2e.png)
So now we just compute
It's obvious that un-normalized,



Okay, so then we get the point



Let





![\[ Q'_x = 1 - \frac{((s-b)+(s-c))(s-a)}{sa} = \frac{sa - a(s-a)}{sa} = \frac{a}{s} \]](http://latex.artofproblemsolving.com/d/f/5/df5679b94f5164297ee9e522f771170be250e4a2.png)
Let


![\[ v=I-Q'=\left(\frac{a}{2s}, \frac{2(s-a)(s-b)-ab}{2as}, \frac{2(s-a)(s-c)-ac}{2acs}\right) \]](http://latex.artofproblemsolving.com/4/0/a/40a49cd58e8fa69bf3e1fcc31055358e35c1f640.png)




\[ \begin{align*} -\frac{1}{4} \cdot \frac{16K^2}{4s^2} &= a^2 \cdot \frac{jk}{(2as)^2} + b^2 \cdot \frac{ak}{4as^2} + c^2 \cdot \frac{aj}{4as^2} \\ &= \frac{jk + b^2k + c^2j}{4s^2} \\ \Leftrightarrow -16K^2 &= 4(jk+b^2k+c^2j) \end{align*} \]
But we also know that

\[ \begin{align*} j &= 2(s-a)(s-b)-ab \\ &= 2s^2-2(a+b)s+ab \\ &= \frac{1}{2} (a+b+c)^2 - (a+b)(a+b+c) + ab \\ &= \frac{1}{2}(a^2+b^2+c^2) + (ab+bc+ca)-(a^2+2ab+b^2+ca+ab)+ab \\ &= \frac{1}{2}(ac^2-a^2-b^2} \end{align*} \]Similarly,
![\[ k=\frac{1}{2} (b^2-a^2-c^2). \]](http://latex.artofproblemsolving.com/5/7/3/573e96c106d6c2883ed7ecc1c3a5511a7e740e2e.png)
So now we just compute
\[ \begin{align*} 4jk + 4b^2k+4c^2j &= (c^2-a^2-b^2)(b^2-a^2-c^2) + 2b^2(b^2-a^2-c^2) + 2c^2(c^2-a^2-b^2) \\ &= (a^2+(b^2-c^2))(a^2+(c^2-b^2)) + 2b^4 + 2c^4 -2a^2b^2 - 2a^2c^2 - 4b^2c^2 \\ &= a^4-(b^2-c^2)^2 + 2b^4 + 2c^4 -2a^2b^2 - 2a^2c^2 - 4b^2c^2 \\ &= a^4+b^4+c^4-2a^2b^2 - 2b^2c^2-2c^2a^2 \\ &= -16K^2 \end{align*} \]as desired.
This post has been edited 1 time. Last edited by v_Enhance, Jan 2, 2012, 7:00 AM